Directions: Simplify each expression by combining like
Terms (pls help if u can )

Directions: Simplify Each Expression By Combining LikeTerms (pls Help If U Can )

Answers

Answer 1

Answer:

1. -y

3. 9a-7

5. 14x-2

7. 7d-6

Step-by-step explanation:

1. 8y-9y=-y

3. 8a-6+a-1

=(8a+a)-(6+1)

=9a-7

5. -x-2+15x

=(-x+15x)-2

=14x-2

7. 8d-4-d-2

=(8d-d)-(4+2)

=7d-6

Answer 2

Answer:

1. -x

3. 9a-7

5. 14x-2

7. 7d-6

Step-by-step explanation:

To do the first one you would just subtract 8 by 9 which is -1 so you would get -x as 1.

To do the third one would would just move the numbers around so you get 8a+a-6-1 and when you simplify you get 9a-7.

To do the fifth one you just have to move the numbers so you get -x+15x-2 and when you simplify you get 14x-2

To do the seventh one you just have to move the numbers so you get 8d-d-4-2 and when you simplify you get 7d-6.


Related Questions

Solving Inequalities and graphing them.

Answers

Answer:

I used to know this not anymore sry g

1/4(8x-4)=
need help please answer

Answers

Answer:

2x-1 is the answer (or if you're trying to multiply it, the answer is -8.)

Step-by-step explanation:

Events A and B are mutually exclusive. Find the missing probability.


P(A) = 1/4 P(B) = 13/20 P(A or B) = ?


4/5


1/2


9/10


3/8

Answers

Answer:

Option C.

Step-by-step explanation:

It is given that,

[tex]P(A)=\dfrac{1}{4}[/tex]

[tex]P(B)=\dfrac{13}{20}[/tex]

It is given that events A and B are mutually exclusive. It means they have no common elements.

[tex]P(A\cap B)=0[/tex]

We know that,

[tex]P(A\ or\ B)=P(A\cup B)=P(A)+P(B)-P(A\cap B)[/tex]

On substituting the values, we get

[tex]P(A\cup B)=\dfrac{1}{4}+\dfrac{13}{20}-0[/tex]

[tex]P(A\cup B)=\dfrac{5+13}{20}[/tex]

[tex]P(A\cup B)=\dfrac{18}{20}[/tex]

[tex]P(A\cup B)=\dfrac{9}{10}[/tex]

Therefore, the correct option is C.

The P (A or B) should be [tex]\frac{9}{10}[/tex]

Given that,

P(A) = 1 by 4  P(B) = 13 by 20

Based on the above information, the calculation is as follows:

[tex]= \frac{1}{4} + \frac{13}{20}\\\\= \frac{5+13}{20} \\\\= \frac{18}{20}\\\\= \frac{9}{10}[/tex]

Learn more: https://brainly.com/question/17429689?referrer=searchResults

PLS SOMEONE HELP ME ON THIS QUESTION!!!!! I WILL GIVE BRAINLIEST TO THE BEST ANSWER!!!!

Answers

Answer:

C.

Step-by-step explanation:

When you replace x by x - h, the graph is shifted h units horizontally.

Here, x is replaced by x - 6.

x - h = x - 6

h = 6

6 is positive, so the shift is 6 units to the right.

Answer: C.

Find the value of x. Round to the nearest tenth.

15.9
12.4
12.8
16.3

Answers

Answer:

x = 15.9

Step-by-step explanation:

Since this is a right triangle, we can use trig functions

cos theta = adj / hyp

cos 28 = 14/x

x  cos 28 = 14

x = 14 / cos 28

x=15.85598

Rounding to the nearest tenth

x = 15.9

ASAP PLEASE GIVE CORRECT ANSWER
In a rectangular coordinate system, what is the number of units in the distance from the origin to the point $(-15, 8)$? Enter your answer

Answers

distance of a point [tex](x,y)[/tex] from origin is $\sqrt{x^2+y^2}$

so distance is $\sqrt{(-15)^2+(8)^2}=\sqrt{225+64}=\sqrt{289}=17$

Answer:

Distance=17 units

Step-by-step explanation:

Coordinates of the origin: (0, 0)

Coordinates of the point in question: (-15, 8)

Distance formula for any two points [tex](x_1,y_1), (x_2,y_2)[/tex] on the plane:

[tex]distance=\sqrt{(x_2-x_1)^2+(y_2-y_1)^2} \\distance=\sqrt{(-15-0)^2+(8-0)^2}\\distance=\sqrt{(15)^2+(8)^2}\\distance=\sqrt{225+64} \\distance=\sqrt{289} \\distance=17[/tex]

(x+3)(x-5)=(x+3)(x−5)=

Answers

Answer:

All real numbers are solutions. 0=0

Step-by-step explanation:

(x+3)(x−5)=(x+3)(x−5)

Step 1: Simplify both sides of the equation.

x2−2x−15=x2−2x−15

Step 2: Subtract x^2 from both sides.

x2−2x−15−x2=x2−2x−15−x2

−2x−15=−2x−15

Step 3: Add 2x to both sides.

−2x−15+2x=−2x−15+2x

−15=−15

Step 4: Add 15 to both sides.

−15+15=−15+15

0=0

All real numbers are solutions.

If P = (3,4), Find: Rx=1 (P)

Answers

Answer:

-1, 4

Step-by-step explanation:

Answer:

Step-by-step explanation:

answer is (-1,4)

Rose likes to water ski using her sister's boat. The boat takes 3 gallons of gas each hour to run. Gas costs $5 per gallon, how much money will she spend on gas to ski for 4 hours?

Answers

Answer:

$60

Step-by-step explanation:

Answer:

60$

Step-by-step explanation:

3 gallons per hr

multiply by 4

12gl  per 4hrs

5 dl per gl

multiply by 12

60 dl per 12 gl

HOPE I HELPED

PLS MARK BRAINLIEST

DESPERATELY TRYING TO LEVEL UP

   ✌ -ZYLYNN JADE ARDENNE

JUST A RANDOM GIRL WANTING TO HELP PEOPLE!

                                  PEACE!

In right triangle ΔABC (m∠C = 90°), point P is the intersection of the angle bisectors of the acute angles. The distance from P to the hypotenuse is equal to 2 in. Find the perimeter of △ABC if AB = 12 in. PLEASE HELP ILL AWARD MORE BRAINLY POINTS

Answers

Answer:

  28 inches

Step-by-step explanation:

The point of intersection of the angle bisectors is the incenter. It is the center of a circle tangent to the three sides of the triangle. The circle has radius 2.

In the attached figure, we have labeled the points of tangency D, E, and F. We know that CE and CF are both of length 2, and we know that the points of tangency are the same distance from an external point where the tangents intersect. That means DA = FA and DB = EB.

The perimeter of the triangle is ...

  P = DA +DB +FA +EB +CF +CE

Using the above relations, this can be written as ...

  P = DA +DB +DA +DB +CF +CE = 2(DA +DB) +2(CE)

We are told that AB is 12 inches, so DA +DB = 12 inches. We also know that CE = 2 inches, so the perimeter is ...

  P = 2(12 in) + 2(2 in) = 28 in

The perimeter of triangle ABC is 28 inches.

Describe how to solve an absolute value equation
*will give brainliest*

Answers

Answer:

Step 1: Isolate the absolute value expression.

Step2: Set the quantity inside the absolute value notation equal to + and - the quantity on the other side of the equation.

Step 3: Solve for the unknown in both equations.

Step 4: Check your answer analytically or graphically.

Step-by-step explanation:

Answer:

Rewrite the absolute value equation as two separate equations, one positive and the other negative

Solve each equation separately

After solving, substitute your answers back into original equation to verify that you solutions are valid

Write out the final solution or graph it as needed

Step-by-step explanation:

Calculate JK if LJ = 14, JM = 48, and LM = 50

Answers

Answer:

JK = 6.86

Step-by-step explanation:

The parameters given are;

LJ = 14

JM = 48

LM = 50

[tex]tan(\angle JML )= \dfrac{Opposite \ leg \ length}{Adjacent \ leg \ length} = \dfrac{LK}{JM} = \dfrac{14}{48} = \dfrac{7}{24}[/tex]

[tex]tan \left( \dfrac{7}{24} \right)= 16.26 ^{\circ }[/tex]

∠JML = 16.26°

Given that ∠JML is bisected by KM, we apply the angle bisector theorem which states that a ray that bisects an interior angle of a triangle bisects the opposite (bisected angle facing side) in the proportion of the ration of the other two sides of the triangle.

From the angle bisector theorem, we have;

LM/JM = LK/JK

50/48 = LK/JK................(1)

LK + KJ = 14.....................(2)

From equation (1), we have;

LK = 25/24×JK

25/24×KJ + JK = 14

JK×(25/24 + 1) = 14

JK × 49/24 = 14

JK = 14×24/49 = 48/7. = 6.86.

JK = 6.86

what seven divided by 4

Answers

Answer:

7 divided by 4 is 1 ¾ as a fraction, or 1.75 as a decimal.

Step-by-step explanation:

Pls mark as brainliest answer

The calculated division of the numbers seven divided by 4 is 1 3/4

How to calculate the division of the numbers

From the question, we have the following parameters that can be used in our computation:

seven divided by 4

When represented as an equation, we have

seven divided by 4 = 7/4

Divide 7 by 4

So, we have the following result

seven divided by 4 = 1 3/4

Using the above as a guide, we have the following:

the result is 1 3/4

Read more about quotient at

brainly.com/question/11418015

#SPJ6

Area of a trapeziod with a top base length of 6 cm a bottom base length of 12 cm and a height of 5 cm​

Answers

Answer:

A=45cm^2

Step-by-step explanation:

A= h/2(b1 + b2) for trapezoid

A=(5/2(6+12))cm^2

A=(2.5(18))cm^2

A=45cm^2

Consider the equation x2+4x+9=0 in standard form. Which equation shows the coefficients a, b, and c correctly substituted into the quadratic formula? Please show all steps to get to the answer, please!!

Answers

Answer:

x = -2+i√5 and  -2i-√5

Step-by-step explanation:

The general form of a quadratic equation is ax²+bx+c = 0

Given the quadratic equation x²+4x+9=0 in its standard form, on comparing with the general equation we can get the value of the constant a, b and c as shown;

ax² = x²

a = 1

bx = 4x

b = 4

c = 9

The quadratic formula is given as x = -b±√(b²-4ac)/2a

Substituting the constant;

x = -4±√(4²-4(1)(9))/2(1)

x = -4 ±√(16-36)/2

x = -4±√-20/2

x = -4±(√-1*√20)/2

Note that √-1 = i

x = -4±(i√4*5)/2

x = (-4±i2√5)/2

x = -4/2±i2√5/2

x = -2±i√5

The solution to the quadratic equation are  -2+i√5 and  -2i-√5

What does the law of cosines reduce to when dealing with a right angle

Answers

Answer:

It is reduced to the equation of the Theorem of Pythagoras.

Step-by-step explanation:

Any triangle can be modelled by this formula under the Law of Cosine:

[tex]b = \sqrt{a^{2}+c^{2}-2\cdot a\cdot c\cdot \cos B}[/tex]

Where:

[tex]a[/tex], [tex]b[/tex], [tex]c[/tex] - Side lengths, dimensionless.

[tex]B[/tex] - Angle opposed to the side [tex]b[/tex], measured in sexagesimal degrees.

Now, let suppose that angle B is a right angle (90º), so that b is a hypotenuse and a and c are legs. Hence:

[tex]\cos B = 0[/tex]

And the equation is reduced to the form of the Theorem of Pythagoras, that is to say:

[tex]b = \sqrt{a^{2}+c^{2}}[/tex]

What is the slope of the line that passes through the points (-10, 8) and
(-15, – 7)? Write your answer in simplest form.

Answers

Answer:

[tex]slope=3[/tex]

Step-by-step explanation:

Use the following equation the find the slope:

[tex]\frac{y_{2}-y_{1}}{x_{2}-x_{1}} =\frac{rise}{run}[/tex]

Rise over run is the change in the y-axis over the change in the x-axis from one point to the other. This is also known as the "slope". Insert the known values:

[tex](-10_{x1},8_{y1})\\\\(-15_{x2},-7_{y2})\\\\\\\frac{-7-8}{-15-(-10)}\\\\\frac{-7-8}{-15+10}[/tex]

Solve:

[tex]\frac{-7-8}{-15+10}=\frac{-15}{-5}[/tex]

Simplify. Two negatives make a positive:

[tex]\frac{-15}{-5}=\frac{15}{5}[/tex]

Simplify fraction by dividing top and bottom by 5:

[tex]\frac{15}{5}=\frac{3}{1} =3[/tex]

The slope is 3.

:Done

The slope of the line that passes through the points (-10, 8) and  (-15, -7) is 3 and thsi can be determined by using the point-slope formula.

Given :

The line that passes through the points (-10, 8) and  (-15, -7).

The following steps can be used in order to determine the slope of the line that passes through the points (-10, 8) and  (-15, -7):

Step 1 - The slope formula when two points are given is:

[tex]\rm m = \dfrac{y_2-y_1}{x_2-x_1}[/tex]

where m is the slope and [tex](x_1,y_1)[/tex] and [tex](x_2,y_2)[/tex] are the points on the line.

Step 2 - Substitute the known terms in the above formula.

[tex]\rm m = \dfrac{-7-8}{-15+10}[/tex]

Step 3 - Simplify the above expression.

[tex]\rm m = \dfrac{15}{5}[/tex]

m = 3

For more information, refer to the link given below:

https://brainly.com/question/2514839

Q.An observer 1.7m tall is 20sqrt(3)m away from a tower.The angle of elevation from the eye of observer to the top of the tower is 30 Find the height of the tower

plz Answer me ​

Answers

Answer:

21.7 m

Step-by-step explanation:

The question above is a right angle triangle and we would be using the trigonometric function of tangent to solve for it.

tan θ = Opposite/ Adjacent

Opposite side = Height = unknown

Adjacent = 20sqrt(3) m

θ = Angle of Elevation = 30°

Hence, we have:

tan 30° = Opposite/ 20√3

Opposite = tan 30° × 20√3m

Opposite = 20m

Height of the tower = Height of the observer + Height (Opposite side)

Height = 20m

Height of the the observer as given in the question is = 1.7m

Height of the tower = 20m + 1.7m

= 21.7m

Therefore, the height of the tower = 21.7m

Please i need to know the meaning of this! : Thanks.

Answers

Come on is your answer

Hope this help




What are the dimensions of the matrix?

Answers

The order of a matrix is m×n where m is the number of rows and n is the number of columns.

can you count and find what are m and n here?

Answer:

Step-by-step explanation:

Number of rows X Number of columns

Rows = 3

Columns = 2

answer = 3x2

In the expression 2^3 the 2 is known as the


base

exponent

rational number

irrational number

Answers

Answer:

Hey there!

In this problem, 2 is known as the base.

Let me know if this helped :)

Answer:

The Base

Step-by-step explanation:

The base in a power is the number being raised to the exponent's power. It is the bigger number in the power.

2 is being raised to the third power. This means that 2 is the base of the power.

3 would be the exponent in the power.

Brainilest Appreciated.

Microsoft Excel might be useful when establishing relationships involving vertex-edge graphs.
O True
False

Answers

Answer:

True

Step-by-step explanation:

Microsoft Excel is a great tool and has saved me on countless occasions in graphs and tables. I agree with the earlier answer by Hedland.


* Graph these numbers on a number line.
-5,3, -2,1
-5

Answers

-5,3,-2,1 on a number line

<-|----|----|----|----|----|----|----|----|->

-5 -2 0 1 3

Ohm's law states That the current That the current (I) In amps equals the voltage (E) In volts divided by the resistance (R) in ohms. If you connected a two megohm resistor (2 x 10 6 ohms) Across a 2.4 kilovolt Voltage source parentheses 2.4Times 10 to the third power volts) What would be the current in amps

Answers

Answer:

1.2 milli amps

Step-by-step explanation:

From ohms law the expression for voltage is given as

[tex]V= IR[/tex]

Now given that

Voltage, V= [tex]2.4*10^3[/tex] volts

Resistance R= [tex]2*10^6[/tex] ohms

Applying ohms law the current can be calculated by making I the subject of formula

that is I= V/R

[tex]I= \frac{2.4*10^3}{2*10^6} \\\\I= \frac{2.4}{2} *10^(^3^-^6^)\\\\I=1.2*10^-^3[/tex]

The current in amps is 1.2 milli amps

what is the angle taken in anticlockwise direction between (1) North-east and south- west​

Answers

Answer:

Step-by-step explanation:

North-east has a bearing of 45 degrees

South-west has a bearing of 225 degrees.

Taken anticlockwise,

angle = south-west - north-east

=  (225-45)

= 180 degrees

answer

hope this helps....

(10 PTS) How do I solve for this? Please show work

Answers

Answer:

4

Step-by-step explanation:

8 ^ 2/3

Rewriting 8 as 2^3

( 2^3) ^ 2/3

We know that a^ b^c = a^ (b*c)

2 ^ ( 3 * 2/3)

2 ^ 2

4

Simplify the following expression.

Answers

Answer:

3x+11y-3

Step-by-step explanation:

Hey! So here is what you do to solve the problem-

Combine like terms:

(x) 5x-2x=3x

(y) 3y+8y=11y

(#) 7-10 =-3

So....

3x+11y-3 is your answer!

Hope this helps!:)

Two trees are leaning on each other in the forest. One tree is 19 feet long and makes a 32° angle with the ground. The second tree is 16 feet long. What is the approximate angle, x, that the second tree makes with the ground? A 0.6° B 35.0° C 39.0° D 58.0°

Answers

Answer:

C 39.0

Step-by-step explanation:

To find the approximate angle, x, that the second tree makes with the ground, we can use the concept of similar triangles Therefore the correct option is B.

Let's calculate the height of the first tree using the given information. We can use the formula for the opposite side in a right triangle: opposite = adjacent * tan(angle). Therefore, the height of the first tree is approximately [tex]19 * tan(32°) = 19 * 0.6249 ≈ 11.873[/tex]  feet. Now, we can set up a proportion between the two trees based on their heights. Let x be the angle the second tree makes with the ground.

We have the following proportion: (height of first tree)/(height of second tree) = (length of first tree)/(length of second tree). Substituting the known values, we have [tex]11.873/16 = 19/x[/tex]. Cross-multiplying gives us [tex]11.873x = 304,[/tex]  and dividing both sides by 11.873 yields[tex]x ≈ 25.63°.[/tex]  The approximate angle, x, that the second tree makes with the ground is closest to 35.0°.

Hence the correct option is B

To know more about triangle visit:

https://brainly.com/question/30966657

#SPJ2

If ∠A and ∠B are vertical angles, what is true about their measures?

Answers

Answer:

they are equal due to the vertical angles theorem

vertical angles theorem states that vertical angles, angles that are opposite each other and formed by two intersecting straight lines, are congruent

Answer:

They are equal

Step-by-step explanation:

I just had it on a exam and can I have brainliest Im trying to level up

I REALLY NEED HELP PLEASE HELP ME :(

Answers

Answer:

I may be wrong but I think 8 is your answer.

Step-by-step explanation:

(-1)^(3/7)  x 128^(3/7)

-1 x 128^3/7

128^(3/7) = 8

= 8

Other Questions
Two coherent sources of radio waves, A and B, are 5.00 meters apart. Each source emits waves with wavelength 6.00 meters. Consider points along the line connecting the two sources.Required:a. At what distance from source A is there constructive interference between points A and B?b. At what distances from source A is there destructive interference between points A and B? The lungs areto the heart.O AnteriorO DistalLateralSuperior On December 31, 2017, Ball Company leased a machine from Cook for a 10-year period, expiring December 30, 2027. Annual payments of $100,000 are due on December 31. The first payment was made on December 31, 2017, and the second payment was made on December 31, 2018. The present value at the inception of the lease for the 10 lease payments discounted at 10% was $676,000. The lease is appropriately accounted for as a capital lease by Ball.Required:1. Compute the December 31, 2018, amount that Ball should report as a lease liability after the first payment has been made. 2. What portion of this total liability should be classified as a current liability? Hey there please help me with this question even in the 21st century some tribes of Nepal are living in hunting and gathering society. Their existence has become critical due to deforestation.In this situation, how should we protect them from extinction? suggestct any four measures. Using these metal ion/metal standard reduction potentials Cd2+(aq)|Cd(s) Zn2+(aq)|Zn(s) Ni2+(aq)|Ni(s) Cu2+(aq)/Cu(s) -0.40 V -0.76 V 0.25 V +0.34 V Calculate the standard cell potential for the cell whose reaction is Ni2+(aq) + Zn(s) Zn2+(aq)+ Ni(s) A woman sold an article for 200 GH cedis and made a profit of 25%. Find the cost price of the article. Question: 4 x 1 2/5 Answer with a mixed number in simplest form! On the statement of cash flows, the cash flows from operating activities section would include a. receipts from the issuance of common stock b. payments for cash dividends c. payment for interest on short-term notes payable d. payments for the purchase of investments The function g is defined as follows for the domain given.g(x) = 2x+1,domain = (-5, -1, 2, 3)Write the range of g using set notation. Then graph g Just wondering if I did this right 3. If the coordinates of the two points 1 pointare P(-7,5) and Q (-6, 9), then(abscissa of P) - (abscissa of Q) is.... *a) 3O b) 1c) -2O d) -1Other: In terms of the trigonometric ratios for ABD, what is the length of line segment BD? Find the probability.Two dice are rolled. Find the probability that the score on the dice is either 5 or10. Help plz and explain if possible mental disorder in the media The basal side of epithelium is always connected to? A) basement membranes B) connective tissue C) epithelial cells D) blood vessels The GoT cups are a fast seller and you need to ensure that you have enough rolls of paper to fulfill demand. The first stage in the process is to determine the total cost of the current inventory ordering model. Given the following information, how many rolls should they order to minimize costs?H: $1.75 per unitD: 500 rolls per monthQ: 100 units ordered at a timeS: $25 per order The IMF policies that accompany most IMF loans are typically: Multiple Choice expansionary in the short run. procyclical in the long run. contractionary in the long run. contractionary in the short run. 8.310x10^2 7.210x10^1[?]x10^[?]